MRCS PART-A ONLINE PREPARATORY COURSE BY NASA KHAN …

42
© Notes by Dr. Sakib (MRCS Online Preparatory Course By Nasa Khan) © Notes by Dr. Sakib (MRCS Online Preparatory Course By Nasa Khan) Page | 1 MRCS PART-A ONLINE PREPARATORY COURSE BY NASA KHAN MOCK EXAM-1 PAPER – 1 (SBA – Single Best Answer) TOTAL MARKS – 180 DURATION – 180 MINUTES (Please use the supplied Look-a-like Answer Sheet) 1. The axillary nerve passes directly inferior to which muscle as it leaves the axilla? A ) Long head of triceps B ) Long head of biceps C ) Pectoralis major D ) Subscapularis E ) Teres minor 2. A 35-year-old man presents to A+E complaining of severe pain in his lower back after lifting a heavy box at work. The pain radiates to his right buttock and thigh. He has had no urinary symptoms. On examination he can straight leg raise to 90 degrees on the left side but only to 30 degrees on the right. Sciatic stretch test is positive. He has difficulty plantarflexing his right ankle and has abnormal sensation on the plantar aspect of the foot. His right ankle reflex is absent but all other reflexes are normal. There is no other sensory disturbance. The likely diagnosis is? A ) Cauda equina syndrome B ) L3/L4 disc prolapse C ) L4/L5 disc prolapse D ) L5/S1 disc prolapse E ) Old Shuerman's disease 3. A 40-year old lady presents to clinic complaining of an 18 month history of dorsoradial wrist pain. She is a keen tennis player. On examination she has tenderness localized to the dorsoradial aspect of the wrist and passive motion of the thumb causes crepitus in the same region. Finkelstein's test is positive. The likely diagnosis is: A ) Carpal tunnel syndrome B ) De Quervain's trenosynovitis C ) Golfer's elbow D) Tennis elbow E) Ulna tunnel syndrome 4. A 46-year-old man presents after penetrating injuries to his arm and forearm. He is unable to extend his fingers. There is no sensory disturbance and there is no vascular injury. Which nerve has been damaged? A ) Anterior interosseous nerve B ) Median nerve C ) Posterior interosseous nerve D ) Radial nerve E ) Ulnar nerve

Transcript of MRCS PART-A ONLINE PREPARATORY COURSE BY NASA KHAN …

Page 1: MRCS PART-A ONLINE PREPARATORY COURSE BY NASA KHAN …

© Notes by Dr. Sakib (MRCS Online Preparatory Course By Nasa Khan)

© Notes by Dr. Sakib (MRCS Online Preparatory Course By Nasa Khan) Page | 1

MRCS PART-A ONLINE PREPARATORY COURSE BY NASA KHAN MOCK EXAM-1

PAPER – 1 (SBA – Single Best Answer)

TOTAL MARKS – 180

DURATION – 180 MINUTES

(Please use the supplied Look-a-like Answer Sheet)

1. The axillary nerve passes directly inferior to which muscle as it leaves the axilla?

A ) Long head of triceps

B ) Long head of biceps

C ) Pectoralis major

D ) Subscapularis

E ) Teres minor

2. A 35-year-old man presents to A+E complaining of severe pain in his lower back after lifting a heavy box

at work. The pain radiates to his right buttock and thigh. He has had no urinary symptoms. On examination

he can straight leg raise to 90 degrees on the left side but only to 30 degrees on the right. Sciatic stretch test

is positive. He has difficulty plantarflexing his right ankle and has abnormal sensation on the plantar aspect

of the foot. His right ankle reflex is absent but all other reflexes are normal. There is no other sensory

disturbance. The likely diagnosis is?

A ) Cauda equina syndrome

B ) L3/L4 disc prolapse

C ) L4/L5 disc prolapse

D ) L5/S1 disc prolapse

E ) Old Shuerman's disease

3. A 40-year old lady presents to clinic complaining of an 18 month history of dorsoradial wrist pain. She is a

keen tennis player. On examination she has tenderness localized to the dorsoradial aspect of the wrist and

passive motion of the thumb causes crepitus in the same region. Finkelstein's test is positive. The likely

diagnosis is:

A ) Carpal tunnel syndrome

B ) De Quervain's trenosynovitis

C ) Golfer's elbow

D) Tennis elbow

E) Ulna tunnel syndrome

4. A 46-year-old man presents after penetrating injuries to his arm and forearm. He is unable to extend his

fingers. There is no sensory disturbance and there is no vascular injury. Which nerve has been damaged?

A ) Anterior interosseous nerve

B ) Median nerve

C ) Posterior interosseous nerve

D ) Radial nerve

E ) Ulnar nerve

Page 2: MRCS PART-A ONLINE PREPARATORY COURSE BY NASA KHAN …

© Notes by Dr. Sakib (MRCS Online Preparatory Course By Nasa Khan)

© Notes by Dr. Sakib (MRCS Online Preparatory Course By Nasa Khan) Page | 2

5. A 29-year-old male presents with symptoms of severe gastro-oesophageal reflux. Which one of the

following is most useful in assessing the role of surgery?

A ) Cardiac sphincter manometry

B ) Gastric emptying study

C ) Intragastric pH monitoring off therapy

D ) Oesophageal motility study

E ) Oesophageal pH monitoring on therapy

6. Which one of the following organs is in direct contact with the anterior surface of the left kidney, without

being separated from it by peritoneum?

A ) Duodenum

B ) Jejunum

C ) Pancreas

D ) Spleen

E ) Stomach

7. A publication assesses a new diagnostic test for thyroid cancer. Which of the following terms would reflect

the number of cases of thyroid cancer correctly identified by this new test?

A ) Accuracy

B ) Negative predictive value

C ) Positive predictive value

D ) Sensitivity

E ) Specificity

8. A 65-year-old woman presented with a malabsorption syndrome. She had a past history of radiotherapy

for cervical cancer. Small intestine biopsy reveals - villous atrophy and crypt hypertrophy, chronic

inflammatory cell infiltrate of the lamina propria together with increase in intra-epithelial lymphocytes. What is

the most likely diagnosis?

A ) Bacterial overgrowth

B ) Coeliac disease

C ) Crohn's disease

D ) Mesenteric ischaemia

E ) Radiation enteropathy

9. A 72-year-old male presented to his GP with depression after the death of his wife. His notes also reveal

that he has a two-year history of urinary hesitancy and poor stream. His GP prescribed him some medication

and the following day he developed acute urinary retention. Which of the following drugs is most likely to

have precipitated the urinary retention?

A ) Amitriptyline

B ) Diazepam

C ) Fluoxetine

D )Venlafaxine

Page 3: MRCS PART-A ONLINE PREPARATORY COURSE BY NASA KHAN …

© Notes by Dr. Sakib (MRCS Online Preparatory Course By Nasa Khan)

© Notes by Dr. Sakib (MRCS Online Preparatory Course By Nasa Khan) Page | 3

E ) Zopicl one

10. Which of the following regarding Helicobacter pylori is false?

A ) Infection may elevate serum gastrin

B ) It is the cause of > 90% of duodenal ulceration

C ) It is associated with gastric adenocarcinoma

D ) Eradication of the infection is curative in gastric MALT lymphoma

E ) It is a Gram positive spiral flagellated organism

11. A study comparing contrast CT colonography with the reference technique of colonoscopy for large

bowel carcinoma reveals the following data in 400 patients:

Investigation CT Positive CT Negative

Colonoscopy Positive 30 10

Colonoscopy negative 20 340

Which one of the following most accurately describes the performance of CT versus colonoscopy for the

diagnosis of large bowel cancer?

A ) There are 340 false negatives

B ) There are 370 false negatives

C ) There are 10 false positives

D ) There are 20 false positives

E ) There are 20 true negatives

12. You are asked to provide advice on a 35-year-old woman who is admitted under the maxillo-facial

surgeons for extraction of wisdom teeth. The only concern was that she had developed prolonged bleeding

following a tooth extraction 10 years previously and had required suturing. Besides this, she gave no other

history of bleeding. What is the most likely diagnosis?

A ) Factor IX deficiency

B ) Factor V Leiden

C ) Factor XII deficiency

D ) Primary antiphospholipid syndrome

E ) Von Willebrand's Disease

13. A 70-year-old woman is referred by a with a breast lump. She was asymptomatic but her investigations

reveal:

Corrected calcium :2.72 mmol/L (2.2 - 2.6)

Phosphate :0.80 mmol/L (0.8-1.4)

Alkaline phosphatase :110 U/L (20 - 95)

PTH concentration :5.1 pmol/L (0.9-5.4)

What is the most likely diagnosis?

A ) Bony metastases

Page 4: MRCS PART-A ONLINE PREPARATORY COURSE BY NASA KHAN …

© Notes by Dr. Sakib (MRCS Online Preparatory Course By Nasa Khan)

© Notes by Dr. Sakib (MRCS Online Preparatory Course By Nasa Khan) Page | 4

B ) Chronic vitamin D excess

C ) Ectopic PTH related peptide (PTHrp) secretion

D ) Multiple myeloma

E ) Primary hyperparathyroidism

14. A 76-year-old gentleman is admitted with a significant lower fresh blood GI bleed. Following endoscopic

examination of upper and lower GI tracts it is established he has bleeding from the sigmoid colon. The bleed

is ongoing and attempt at radiological embolization is carried out.Which of the following statements best

describes the anatomy of the vessel needing embolization?

A) An anterior branch of the aorta arising at the level of L1

B) An anterior branch of the aorta arising at the level of L3

C) Lateral branch of the aorta arising at Ll

D) Llateral branch of the aorta arising at L3

E) Lateral branch of the aorta arising at T12

15. A 69-year-old gentleman presents with loin pain and frank haematuria. After investigation he is

diagnosed with a renal cell carcinoma. Which is the most common mode of initial dissemination of renal cell

carcinoma?

A) Along the renal vein

B) Multiple lung metastases that regress after resection of the primary

C) To the IVC

D) To the ureter

E) Solitary lung metastases

16. A 76-year-old man presents with weight loss, dark urine, and pale stools which are difficult to flush away.

An excess of which of the following would account for this history?

A) Conjugated bilirubin

B) Hyperbilirubinaemia

C) Stercobilinogen

D) Unconjugated bilirubin

E) Urobilinogen

17. A 28-year-old female presents eight weeks post-partum with a painful, swollen, erythematous right

breast. On examination there is fluctuance in the right upper outer quadrant adjacent to nipple areolar

complex. The overlying skin is red and tender. What is the most appropriate management for this patient?

A) Excision of abscess cavity

B) Incision and drainage

C) Intravenous antibiotics

D) Mammograghic guided needle aspiration

E) Ultrasound guided needle aspiration

18. This 16-year-old male is admitted with abdominal pain following an accident on a motor bike. On

examination, he has marked abdominal tenderness, a pulse of 102 bpm and a blood pressure is 100/60

mmHg. Which of the following is the likely diagnosis?

Page 5: MRCS PART-A ONLINE PREPARATORY COURSE BY NASA KHAN …

© Notes by Dr. Sakib (MRCS Online Preparatory Course By Nasa Khan)

© Notes by Dr. Sakib (MRCS Online Preparatory Course By Nasa Khan) Page | 5

A) Aortic rupture

B) Laceration of the kidney

C) Liver laceration

D) Ruptured duodenum

E) Splenic haematoma

19 . A 60-year-old is complaining of facial pain. Sinus X-rays were performed followed by a CT.What is the

diagnosis?

A) Cortical osteoma

B) Fracture of the frontal b one

C) Mucocele

D) Osteogenic sarcoma

E) Osteoid osteoma

20. A50-year-old complained of a fluctuant swelling on the left side of the face over the parotid region. An

ultrasound examination was requested.

A) Arterio-venous malformation.

B) Lipoma

C) Parotid cyst.

D) Pleomorphic adenoma.

E) Warthin's tumour.

21. A 25-year-old is admitted with severe abdominal pain. He had been stabbed 2 years previously. What is

the diagnosis?

A) Aortic transection

B) Partial rupture of the left hemi-diaphragm

C) Pleural haematoma

D) Pneumonia

E) Splenic haematoma

22. A patient requires a blood transfusion following joint replacement surgery. They ask you about the risks

of acquiring a transfusion related infection. Which of the following infections is currently tested for?

A) Hepatitis A

B) Human T-cell leukaemia virus (HTLV)

C) Malaria

D) Syphilis

E) Variant Creutzfeldt-Jakob disease (vCJD).

23. A 65-year-old man has locally advanced pancreatic cancer and has been paying privately for treatment

with Erlotinib (Tarceva) for the past 9 months. It has worked effectively for that period but a recent CT scan

showed further growth in the tumour. Which of the following mechanisms best explains this resistance to

treatment with Erlotinib

A) Development of antibodies to Erlotinib

Page 6: MRCS PART-A ONLINE PREPARATORY COURSE BY NASA KHAN …

© Notes by Dr. Sakib (MRCS Online Preparatory Course By Nasa Khan)

© Notes by Dr. Sakib (MRCS Online Preparatory Course By Nasa Khan) Page | 6

B) lack of autophosphorylation at binding site

C) Malabsorption

D) Mutated IGF- 1 receptor

E) Reduced expression of EGFR

24. A 33-year-old type 1 diabetic male presents with a two day history of pain in his left hand, fever and a

rash. This began after he pricked his finger in the garden whilst pruning a bush. His diabetic control has been

quite reasonable with a HbA1c of 7.1% (3.8-6.4) on basal bolus insulin consisting of Lispro tds and Humulin I

in the evenings. On examination he has a painful, red and swollen middle finger with the redness extending

to the metacaropophalangeal joint. He is diagnosed with cellulites and is prescribed antibiotics. What is the

most likely infective organism?

A) Escherichia coli

B) Klebsiella

C) MRSA

D) Pseudomonas aeruginosa

E) Streptococcus pyogens

25. 'Which of the following statement regarding the internal Jugular vein and relations is true'?

A) Originates at the carotid canal

B) Lies lateral to the common carotid artery

C) Passes posterior to the subclavian artery

D) Passes behind the clavicle to join the Superior vena cava

E) Passes in front of the clavicle to join the Superior vena cava

26. A 10-year-old boy of Asian descent complains of pain in the pelvis which is exacerbated by movements.

He has lost some weight over the last one month. What is the likely diagnosis?

A) Ewing's tumour

B) Osteomalacia.

C) Osteogenic sarcoma

D) Osteoid osteoma

E) Osteomyelitis

27. A firm 2-3 cm mass is palpable in the upper outer quadrant of the right breast of a 52-year-old woman.

There are no palpable axillary lymph nodes. A lumpectomy with axillary node dissection is performed and the

breast lesion is found to have positive immunohistochemical staining for HER2/neu (c-erb B2). Staining for

oestrogen and progester one receptors is negative. Which of the following additional treatment options is

most appropriate, based upon these findings?

A) Radical mastectomy

B) St John's wort

C) Tamoxifen

D) Trastuzumab

E) Vancomycin

Page 7: MRCS PART-A ONLINE PREPARATORY COURSE BY NASA KHAN …

© Notes by Dr. Sakib (MRCS Online Preparatory Course By Nasa Khan)

© Notes by Dr. Sakib (MRCS Online Preparatory Course By Nasa Khan) Page | 7

28. Carcinoid tumours of the lung (bronchial adenomas) originate from which of following cell types?

A) Ciliated cell

B) Clara cell

C) Kulchitsky (K) cell

D) Mucus (goblet) cell

E) Type 2 Alveolar cell

29. An experienced group of surgeons report on a randomised placebo-controlled trial comparing a particular

carotid surgery technique as compared to a sham operation. Their study concludes that 'using this advanced

surgical technique reduces the risk of stroke from 4.3% to 3.8% (p<0.05)'.What has this study proved about

the surgical Procedure?

A) Acceptability

B) Effectiveness

C) Efficacy

D) Safety

E) Usefulness

30. A 68-year-old female with terminal bowel cancer is receiving optimal doses of morphine sulphate

therapy. Which of the following effects may be expected with the addition of a partial opioid agonist?

A ) Increased analgesia

B ) Increased respiratory depression

C ) Increased sedation

D ) No change

E ) Reduced analgesia

31. A 30-year-old woman has a right mastectomy and axillary lymph node dissection for a carcinoma

diagnosed by fine needle aspiration cytology. The histological pattern is that of a poorly differentiated

carcinoma that is negative for oestrogen and progester one receptors, but is positive for HER2/neu- one

axillary lymph node demonstrates micrometastases. Her 32-year-old sister is found to have a similar lesion.

Which of the following statements regarding risk factors for this lesion is the most appropriate?

A) A history of late menarche is likely to be present in females in this family

B) Fibrocystic changes were present for many years

C) She had a history of exposure to hydrocarbon compounds

D) She has a positive antinuclear antibody test

E) These findings suggest a BRCA-1 mutation

32. A 64-year-old man is admitted with central epigastric pain. Abdominal X-ray shows a central dilated

bowel loop. His temperature is 37.O°C, pulse 130 beats per min, blood pressure 80/50 mmHg, respiratory

rate 29/min and SpO2 90 on air. His Full blood count reveals:

Haemoglobin :13.0 g/dL (13.0-18.0)

White Cell Count :3.2 x 109/L (4-11 x 10

9)

Platelets :108 x 109/L, (150-400 x 10

9)

Page 8: MRCS PART-A ONLINE PREPARATORY COURSE BY NASA KHAN …

© Notes by Dr. Sakib (MRCS Online Preparatory Course By Nasa Khan)

© Notes by Dr. Sakib (MRCS Online Preparatory Course By Nasa Khan) Page | 8

MCV :105 fL (80-96)

Which of the following is the most likely diagnosis?

A) Gall st one ileus

B) Ischaemic bowel

C) Pancreatitis

D) Perforated duodenal ulcer

E) Small bowel obstruction

33. A 56-year-old male presents with generalised fatigue and upper abdominal discomfort with some weight

loss over the last six months. He has otherwise been well but admits to consuming 10 units of alcohol per

day.On examination you note 2cm hepatomegaly. Liver function tests show :

ALT :140 IU/L (5-35)

AST :150 IU/L (1-31)

Alkaline Phosphatase : 250 IU/L (45-105)

MCV : 110 fL (80-96)

Which of the following is the most likely cause of his presentation?

A) Alcoholic hepatitis

B) Chronic active hepatitis

C) Hepatoma

D) Metastatic liver disease

E) Viral hepatitis

34. A 6-year-old female presents to the emergency department in status epilepticus of 30 minutes duration.

The airway is maintained using an oropharyngeal airway and high flow oxygen is delivered. Multiple attempts

at venous cannulation are unsuccessful. The blood glucose (BM) is 5.2 mmol/L (3.0-6.0). Which of the

following therapies would be most appropriately used in this patient to terminate the seizure activity?

A) Lorazepam

B) Midazolam

C) Paraldehyde

D) Phenytoin

E) Thiopental

35. A 24-year-old female teacher collapsed in the classroom after complaining of a severe headache

associated with nausea. She had been well in the preceding few days. She had no past medical history and

did not take any regular or illicit drugs. She did not smoke or drink.On arrival in Casualty Department, her

Glasgow coma scale was 12/15 (motor 6 vocal 3 eyes 3) and there was nuchal rigidity. Her blood pressure

was 145/85 mmHg, pulse was 90 beats per minute and regular and temperature was 37. 1°C Heart sounds

were normal and chest appeared clear. Cranial nerve examination revealed no abnormalities. Fundoscopy

was normal. There were no obvious focal neurological signs on examining the peripheral nervous system,

although both plantar responses were extensor.

Investigations revealed:

Haemoglobin :12.5 g/dL (13.0-18. 0 g/dL)

White cell count :11. 0 x 109/L (4-11 x 10

9/L)

Platelets :267 x 109/L (150-400 x 10

9/L)

Page 9: MRCS PART-A ONLINE PREPARATORY COURSE BY NASA KHAN …

© Notes by Dr. Sakib (MRCS Online Preparatory Course By Nasa Khan)

© Notes by Dr. Sakib (MRCS Online Preparatory Course By Nasa Khan) Page | 9

Serum sodium :142 mmol/L (137-144 mmol/L)

Serum potassium :3.9 mmol/L (3.5-4.9 mmol/L)

Serum urea :5.1 mmol/L (2.5-7.5 mmol/L)

Serum creatinine :94 mol/L (60-110 mol/L)

Serum CRP :5 U/L (<5 U/L)

Fasting plasma glucose :5.4 mmol/L (3-6 mmol/L)

Serum cholesterol :3.8 mmol/L (< 5.2 mmol/L)

A CT scan of her brain was normal

A lumbar puncture was performed and yielded the following data:

Appearance Straw-coloured fluid

Opening pressure :15cmH2O (6-18 cmH20)

White cell count :6 cells/mL (<5 /mL)

Red cell count :1450 cells/mL (<5 / mL)

Protein :0.46 g/L (0. 15-0.45 g/L)

What should be the next step in the management of this patient?

A) Arrange a four-vessel cerebral angiogram

B) Arrange an MR angiogram

C) Arrange urgent MRI brain scan

D) Commence on intravenous cefotaxime and acyclovir

E) Refer to neurosurgeons for urgent assessment

36. A 41-year-old male presents with an acute onset of epigastric pain. His serum amylase on admission is

1874 U/I (NR 60-180) and he is diagnosed with acute pancreatitis. Which of the following parameters form

part of the Modified Glasgow criteria for stratifying acute pancreatitis?

A) Alkaline phosphatase

B) C Reactive protein

C) Creatinine

D) Glucose

E) PaCO2

37. A 38-year-old female is admitted as an emergency with severe right upper quadrant pain and vomiting.

On examination she is pyrexial and palpation reveals tenderness and peritonism at the level of the tip of the

9th rib. Investigations reveal:

Bilirubin : 9 µmol/L (1-22)

Amylase : 50 IU/L (50-130)

Haemoglobin : 12.8 g/dL (11.5-16.5)

Platelets :172 x 109/L (150-400 x 10

9)

White ceil-count :15 x 109/L (4-11 x10

9)

What is the likely diagnosis?

A) Acute cholecystitis

B) Acute pancreatitis

C) Ascending cholangitis

D) Biliary colic

Page 10: MRCS PART-A ONLINE PREPARATORY COURSE BY NASA KHAN …

© Notes by Dr. Sakib (MRCS Online Preparatory Course By Nasa Khan)

© Notes by Dr. Sakib (MRCS Online Preparatory Course By Nasa Khan) Page | 10

E) Obstructive jaundice

38. A 74-year-old female presents with an acute onset of night upper quadrant pain. On examination,

temperature is 38.5°C, pulse 115 beats per minute and blood pressure is 96/55 mmHg. Examination of her

abdomen reveals right upper quadrant tenderness. Investigations reveal:

Haemoglobin : 12.6 g/dL (11.5-16.5)

White cell count : 26.5 x 109/L (4-11 x 10

9)

Platelets : 165 x 109/L (150-400 x 10

9)

C-reactive protein : 316 mg/dL (< 10)

Abdominal X-ray : Air in the biliary tree

What is the most likely diagnosis?

A) Biliary colic

B) Cholangitis

C) Cholecystitis

D) Cholelithiasis

E) Gallst one ileus

39. A 35-year-old shop worker presents with pain in her calves which develops after 50 yards of walking. The

pain settles with rest. On examination she has yellow discolouration of her palmar creases. Her fasting lipid

profile reveals: Cholesterol 9.6 mmol/l (<5) Triglycerides 7.3 mmol/l (<2). What is the likely diagnosis?

A) Chylomicronaemia

B) Familial hyperlipidaemia

C) Hypoalphalipoproteinaemia

D) Type III hyperlipidaemia

E) Type IV hyperlipidaemia

40. A 49-year-old female 2 days post total thyroidectomy has developed tetany in both hands. Her corrected

serum calcium is 1.90 mmol/L (2.2-2.6). What is the most appropriate management for this patient?

A) Breathing into a brown paper bag

B) Intravenous calcium gluconate

C) Oral calcium supplements

D) Oral thyroxine

E) Surgical re-exploration of the neck and revascularisation of the paprathyroids

41. A 69-year-old male is seen in Outpatients. He reports weight loss of 1 st one over 3months but his

history is other wise unremarkable. On examination his abdomen is softwith no palpable masses. A PR

examination is normal. His blood tests show:

Haemoglobin : 8.0 g/dL (12-16)

MCV :70 fL (80-96)

Which of the following is the most appropriate investigation for this patient?

A) Abdominal X-ray and colonoscopy

B) CT scan of the abdomen and upper GI endoscopy

Page 11: MRCS PART-A ONLINE PREPARATORY COURSE BY NASA KHAN …

© Notes by Dr. Sakib (MRCS Online Preparatory Course By Nasa Khan)

© Notes by Dr. Sakib (MRCS Online Preparatory Course By Nasa Khan) Page | 11

C) Sigmoidoscopy upper GI endoscopy

D) Ultrasound scan of abdomen and colonoscopy

E) Upper GI endoscopy and colonoscopy

42. A 38-year-old female presents with colicky night upper quadrant pain and jaundice and has noted dark

urine and pale stool for 3 days. She has had several previous episodes of pain associated with fatty meals

and on admission, her bilirubin is 120 µmol/L (1 -22) and alkaline phosphatase is 550 IU/L (45-105), The

remainder of her LFTs are normal. What is the most likely diagnosis?

A) Choledocholithiasis

B) Gilbert's syndrome

C) Pancreatic cancer

D) Primary sclerosing cholangitis

E) Viral hepatitis

43. An 85-year-old man weighing 80 kg is admitted as an emergency with lower abdominal pain and fever.

He is delirious, has a temperature of 39.0°C and has a blood pressure of 80/40mmHg. Urinalysis is strongly

positive for blood protein and nitrates. The microbiologist recommends the prescription of gentamycin. Which

of the following is the most appropriate dose of gentamycin to initiate for this man?

A) 40 mg single dose

B) 40 mg once daily

C) 80 mg 8 hourly

D) 80 mg 12 hourly

E) 160 mg single dose

44. Which of the following has been shown to increase the risk of prostate cancer?

A) Caucasian race

B) Exposure to cadmium

C) Family history of colon cancer

D) Low intake of animal fats

E) Occupational exposure to dust

45. A 36-year-old female who is on warfarin after suffering a deep vein thrombosis, presents with an INR of

8.2 (INR <1.4) and a conjunctival haemorrhage. The blood pressure is 125/55 mmHg, heart rate is 65 bpm

and the ECG reveals a normal sinus rhythm.Which of the following is the most appropriate treatment for this

patient?

A) FFP

B) Factor VII

C) Oral vitamin K 1 mg

D) Prothrombin complex concentrate

E) Stop warfarin only

46. A 78-year-old female who is on warfarin for atrial fibrillation presents with melaena. The blood pressure is

90/60 mmHg and the heart rate 100 bpm. Investigations show:

Page 12: MRCS PART-A ONLINE PREPARATORY COURSE BY NASA KHAN …

© Notes by Dr. Sakib (MRCS Online Preparatory Course By Nasa Khan)

© Notes by Dr. Sakib (MRCS Online Preparatory Course By Nasa Khan) Page | 12

Haemoglobin :9 g/L (12-16)

MCV :87fL (83-95)

INR :7.2 (<1.4)

A PR examination confirms melaena. Which is the best option for correcting the coagulopathy?

A) FFP

B) IV Vitamin K

C) Stop warfarin and give IV Vitamin K

D) Stop warfarin and give IV Vitamin K and Prothrombin complex concentrate

E) Stop warfarin

47. A 36-year-old female intravenous drug abuser is seen in the A&E department with jaundice a fever and

vague right upper quadrant pain. Prior to accepting the patient, a liver function test is requested and this

shows:

Bilirubin :170 µmol/L (1-22)

Aspartate aminotransferase :1500IU/L(1-31)

Alkaline phosphatase :220 IU/L (45-105)

What is the most likely diagnosis?

A) Acute cholecystitis

B) Choledocholithiasis

C) Hepatitis B

D) Pancreatic cancer

E) Sclerosing cholangitis

48. A 14-year-old boy attends the clinic with a first episode of painless jaundice after a bout of

gastroenteritis. His bilirubin is elevated at 78 µmol/l, (1 -22) but the remainder of the liver function was

normal. Interestingly, his reticulocyte count is 20%. What is the likely diagnosis?

A) Biliary atresia

B) Choledocholithiasis

C) Gilbert's syndrome

D) Hepatoblastoma

E) Viral hepatitis

49. A cohort study of 7,500 patients aimed to find out whether the use of olive oil in cooking has an impact

on cardiovascular disease. Approximately half the patients used olive oil in cooking and half used animal fat.

Which of these is a disadvantage of a cohort study?

A) It is not possible to measure the incidence/risk of a disease

B) They are susceptible to recall bias, there is a differential ability of patients to remember exposure

to a risk factor

C) They are not suitable when exposure to risk factors is rare

D) They can only provide information about one outcome

E) When the outcome of interest is rare a very large sample size is needed.

50. A 79-year-old gentleman is admitted with a history of nocturia, frequency, poor flow, hesitancy and

terminal dribbling, and retention due to benign prostatic hypertrophy is diagnosed. Insertion of a catheter

Page 13: MRCS PART-A ONLINE PREPARATORY COURSE BY NASA KHAN …

© Notes by Dr. Sakib (MRCS Online Preparatory Course By Nasa Khan)

© Notes by Dr. Sakib (MRCS Online Preparatory Course By Nasa Khan) Page | 13

drains 2000 mL. Whilst awaiting urological review, a check U&E has shown that his sodium level has

increased from 138 mmoL/L on admission to 155 mmol/L (137-144) and his potassium is unchanged at 3.7

mmol/L (3.5-4.9). He has been normotensive throughout his admission. What is the likely cause of this

finding?

A) Cardiac failure

B) Cirrhosis

C) Cushing's syndrome

D) Nephrotic syndrome

E) Post-obstruction diuresis

51. A trial is proposed to see whether excess alcohol use is a risk factor for osteoporosis. It is decided to

perform a case-control study rather than a cohort study. Which of these is an advantage of a case-control

study?

A) It can provide information on a wide range of outcomes

B) It is possible to measure the incidence of a disease directly

C) It is possible to study exposure to factors that are rare

D) It is relatively quick, cheap and easy to perform

E) The time sequence of events can be assessed

52. You intend to undertake a study of patients who have underg one excision of minor lumps and bumps

over the last 5 years. Specifically you wish to compare post-operative infection rates and also whether there

are any differences between the clinical diagnosis and the histological diagnosis between the differing

grades of practiti oner performing the procedure.Which of the following statements are correct concerning

this study?

A) Approval for the study must be obtained from the local ethics committee.

B) If a study had already published with exactly the same concept then this constitutes plagiarism.

C) If such a study has already been published then the investigators must obtain consent from the

original authors to replicate their work.

D) This is an audit and does not require ethical committee approval.

E) The study is flawed in its design and should not be performed.

53. A 30-year-old male becomes anuric following abdominal surgery. Investigations reveal:

Serurn urea :9.2 mmol/L (2.5-7.5)

Serum creatinine :120 µmol/L (60- 110)

What is the next single most appropriate course of action in. the management of the anuria?

A) 1L normal saline intravenously over 1 hour

B) Commence intravenous low-dose dopamine

C) Give 500ml intravenous plasma expander

D) Give an 80mg intravenous bolus of Furosemide

E) Urgent renal ultrasound

54. A 22-year-old female is admitted following severe injuries sustained in a road traffic accident. She is

communicative but in shock with low blood pressure and tachycardia. You realise that without a transfusion

she will die but she informs you that she has recently become a Jehovah's Witness and that she adamantly

refuses transfusion despite knowledge that she could die. Her distraught parents tell you that she has only

Page 14: MRCS PART-A ONLINE PREPARATORY COURSE BY NASA KHAN …

© Notes by Dr. Sakib (MRCS Online Preparatory Course By Nasa Khan)

© Notes by Dr. Sakib (MRCS Online Preparatory Course By Nasa Khan) Page | 14

recently joined the Jehovah's Witnesses and implore you to transfuse her, as they insist that she does not

know her own mind. Together with other intervention which she permits, what is the most appropriate action

regarding possible transfusion?

A) Declare her incompetent and transfuse.

B) Do not transfuse even if It means that she will die.

C) Get immediate psychiatric intervention to section her and then transfuse.

D) Transfuse immediately, irrespective of the patient's wishes.

E) Wait until she becomes unconscious and then get consent from her parents to transfuse.

55. You are asked to provide advice on a 35-year-old woman who is admitted under the maxillo-facial

surgeons for extraction of wisdom teeth. The only concern was that she had developed prolonged bleeding

following a tooth extraction 10 years previously and had required suturing. Besides this, she gave no other

history of bleeding. What is the most likely diagnosis?

A) Factor IX deficiency

B) Factor V Leiden

C) Factor XII deficiency

D) Primary antiphospholipid syndrome

E) Von Willebrand's Disease

56. A 73-year-old lady presents with abdominal pain and distension. She claims to having not opened her

bowels in the last week or passing flatus for 2 days. On examination, she appears slow and has a

temperature of 35.5°C. Abdominal x-ray shows dilated loops of small and large bowel.What is the most likely

cause of her presentation?

A) Colitis

B) Colonic cancer

C) Diverticulitis

D) Pseudo obstruction

E) Small bowel obstruction

57. A 23-year-old female is seen in the accident and emergency department. She has suffered severe

vomiting and diarrhoea over the last four days. She states that she has been unable to keep down any fluids.

Clinically she is dehydrated and she is started on an intravenous infusion. Investigations reveal a potassium

of 2.6 mmol/L (3.5-4.9). Which of the following abnormalities would you expect to see on her ECG?

A) Shortened P-R interval

B) Small P waves

C) S-T segment depression

D) Tall, tented T waves

E) Wide QRS complexes

58. This 55-year-old male presents with these lumps in his groin which have been present for about one

year and are now aching a little. He describes that on lying flat they entirely disappear. On examination

whilst lying flat no masses are palpable or seen. On standing these masses are seen and are soft and

fluctuant to palpation.What is the likely diagnosis?

A) Direct inguinal hernia

B) Femoral hernia

C) Indirect inguinal hernia

Page 15: MRCS PART-A ONLINE PREPARATORY COURSE BY NASA KHAN …

© Notes by Dr. Sakib (MRCS Online Preparatory Course By Nasa Khan)

© Notes by Dr. Sakib (MRCS Online Preparatory Course By Nasa Khan) Page | 15

D) Lipoma

E) Saphena varix

59. During a neurological examination of the upper limb you attempt to elicit the triceps reflex. You place the

patients arm across the chest, with the elbow flexed at 90°. The triceps tendon is struck with the tendon

hammer.Which nerve (and its nerve root) are you testing?

A) Median nerve C6

B) Median nerve C7

C) Radial nerve C5

D) Radial nerve C6

E) Radial nerve C7

60. A 44-year-old lady with a history of alcohol abuse is brought to A + E after a large haematemesis. Fluid

resuscitation is started, but it appears that bleeding is ongoing. An emergency endoscopy is arranged. The

gastroenterologist notes that she is bleeding from varices in the lower part of her oesophagus. Which vein

drains this area into the portal system?

A) Azygous vein

B) Inferior mesenteric vein

C) Left gastric vein

D) Lower oesophageal vein

E) Thoracic vein

61. Which of the following is true of the musculocutaneous nerve:

A) Is a branch of the lateral cord of the brachial plexus.

B) Leaves the plexus at the point at which it crosses the first rib.

C) Receives fibres from the 8th cervical spinal root.

D) Only contains sensory fibres.

E) None of the above

62. A 35-year-old woman presents as an emergency on the medical intake complaining of a right sided

pleuritic chest pain and shortness of breath. She had underg one a laparoscopic cholecystectomy a week

previously during which there was an iatrogenic tear in the gallbladder during mobilisation. On examination

she had a temperature of 38.2°C, a pulse of 96 bpm and respiratory rate of 24/min. There was tenderness

on palpation and dullness to percussion over the right fib margin and decreased air entry at the right lung

base. What is the diagnosis?

A) Aspiration

B) Atelectasis

C) Pneumonia

D) Pneumothorax

E) Subphrenic Abscess

63. A 64-year-old man is admitted with central epigastric pain. Abdominal X-ray shows a dilated bowel loop.

His temp. is 37.0°C, pulse 130 bpm, blood pressure 80/50 mmHg, respiratory rate 29/min, SpO2 90% on air.

His full blood count reveals:

Page 16: MRCS PART-A ONLINE PREPARATORY COURSE BY NASA KHAN …

© Notes by Dr. Sakib (MRCS Online Preparatory Course By Nasa Khan)

© Notes by Dr. Sakib (MRCS Online Preparatory Course By Nasa Khan) Page | 16

Haemoglobin :13.0g/dL (13.0-18.0)

White cell count :3.2 x 109/L (4-11 x 10

9)

Platelets :108 x109/L (150-400 x 10

9)

MCV :105 fL (80-96)

Which of the following is the most appropriate treatment of this patient?

A) 100% oxygen by rebreather

B) 2x 14 gauge venflons and 2 litres Hartmanns

C) Intensive care

D) Intubation and ventilation

E) Invasive monitoring

64. Your crash beep alerts you to a road traffic accident victim that has just arrived in the A&E department. In

the casualty is am 18-year-old man who is brought in by the paramedics following a RTA. He is opening his

eyes to painful stimuli, his speech is inappropriate, and he is extending his limbs to pain.What is his Glasgow

coma score?

A) 4

B) 6

C) 7

D) 8

E) 10

65. A 56-year-old man is admitted with epigastric pain after drinking heavily. He has a temperature of

36.9°C, a pulse of 95/min, a blood pressure of 85/60 mmHg, and a respiratory rate 32/min. Investigation

reveal:

Haemoglobin :12.6 g/dL (13.0-18.0)

Platelets :169 x 109/L (150-400 x 10

9)

White cell count :3.9 x 109/L (4-11 x 10

9)

Which of the following is the diagnosis?

A) Leaking aortic aneurysm

B) Multi-organ dysfunction syndrome.

C) Pancreatitis

D) Septic shock

E) Systemic Inflammatory Response Syndrome

66. An otherwise fit 80-year-old lady presents with an 8 hour history of acutely ischaemic left upper limb.

There was no history of pain or restricted movement prior to this current episode. She has a history of atrial

fibrillation for which she has been prescribed digoxin and takes appropriate medications for hypertension. On

examination the hand is cold but movement and sensation are preserved. There is a good axillary artery

pulse but distal pulses are impalpable.What is the correct management?

A) Arrange a Duplex scan

B) Book an Outpatient Appointment for Vascular Surgery

C) Commence Heparin

D) Embolectomy

E) Thrombolysis

Page 17: MRCS PART-A ONLINE PREPARATORY COURSE BY NASA KHAN …

© Notes by Dr. Sakib (MRCS Online Preparatory Course By Nasa Khan)

© Notes by Dr. Sakib (MRCS Online Preparatory Course By Nasa Khan) Page | 17

67. A 66-year-old male with a history of ischaemic heart disease and peripheral vascular disease presents

with acute onset generalised abdominal pain and bloody diarrhoea. What diagnosis is indicated by this

contrast study?

A) Carcinoma of transverse colon

B) Crohn's disease

C) Diverticular disease

D) Infective colitis

E) Ischaemic colitis

68. An 85-year-old lady falls whilst out shopping and injures her left upper limb. She attends A&E and an X-

Ray confirms a fracture of the shaft of her humerus. Whilst being assessed in casualty it is noted that her

forearm pulses are weak on the side of the fracture. Which artery is she likely to have damaged?

A) Axillary

B) Brachial

C) Radial

D) Subclavian

E) Ulnar

69. A 35-year-old woman is referred as an emergency with severe retrosternal and upper abdominal chest

pain. On taking a more detailed history it becomes evident that symptoms have been getting progressively

worse for 18 months and the pain is associated with progressive dysphagia, and weight loss of 2 st ones. An

endoscopy demonstrated a dilated oesophagus containing a large volume of food debris but no pathology

was observed. A contrast study was then performed. What is the likely diagnosis?

A) Achalasia

B) Angina

C) Carcinoma of the Oesophagus

D) Gastric Ulcer

E) Reflux Oesophagitis

70. A 21-year-old male is admitted with acute onset headache and is drowsy. He is opening his eyes

spontaneously, is disoriented but is localizing to painful stimuli. Which of the following is the investigation of

choice for this man?

A) Computed Tomography (CT)

B) Lumbar puncture (LP)

C) Magnetic resonance angiography (MRA)

D) Magnetic resonance imaging (MRI)

E) Positron Emission Tomography (PET)

71. During an orthopaedic examination of the hip you carry out, the following examination, The patient

stands directly in front of you. You place yours hands on their anterior superior iliac spines. You then ask

them to lift their good leg off the floor so that their weight is on the bad leg that is being examined. You note

which way the pelvis tilts. What is the name of this test?

A) Lachmann test

B) Macintosh test

C) McMurray test

D) Thomas test

Page 18: MRCS PART-A ONLINE PREPARATORY COURSE BY NASA KHAN …

© Notes by Dr. Sakib (MRCS Online Preparatory Course By Nasa Khan)

© Notes by Dr. Sakib (MRCS Online Preparatory Course By Nasa Khan) Page | 18

E) Trendelenberg test

72. During a neurological examination of the lower limb you attempt to elicit the knee reflex. You place your

hand under the knee, then strike the knee just below the patella whilst you watch the quadriceps muscle.

What nerve roots and nerve are involved in this reflex?

A) Femoral nerve L2, L3

B) Femoral nerve L3, L4

C) Sciatic nerve L2, L3

D) Sciatic nerve L3. L4

E) Sciatic nerve L4, L5

73. During a neurological examination of the lower limbs, you test the ankle reflex. You hold the foot in

external rotation. The knee is flexed. You strike the Achilles tendon with your tendon hammer whilst watching

the calf. Which nerve is involved and what is its nerve root?

A) Deep peroneal nerve L5, S1

B) Deep peroneal nerve S1, S2

C) Tibial nerve L5, S1

D) Tibial nerve S1,,S2

E) Tibial nerve S2, S3

74. An unconscious patient is brought into A + E as an emergency. He has been assaulted with a hammer

by an unknown individual. An emergency CT scan reveals an extradural haematoma. The CT reveals a

fracture through the night pterion.Which blood vessel has bled?

A) Anterior branch of the middle meningeal artery

B) Maxillary artery

C) Middle cerebral artery

D) Posterior branch of the middle meningeal artery

E) Posterior auricular artery

75.The following conditions all can cause a pleural effusion.Which of the following would typically cause a

transudate (<30 mg/L protein)?

A) Metastatic pancreatic cancer

B) Mesothelioma

C) Nephrotic syndrome

D) Pneumonia

E) Sub-phrenic abscess

76. A 32-year-old lady is brought to the neurosurgical ward after a subarachnoid haemorrhage is confirmed

on CT scan. A small, middle cerebral artery aneurysm is subsequently embolised. She later becomes drowsy

and less responsive. A repeat CT head shows hydrocephalus and high cerebro spinal fluid (CSF) pressure is

confirmed with an opening pressure of 26 cmH2O on lumber puncture. Roughly what is the normal circulating

volume of CSF in an adult?

Page 19: MRCS PART-A ONLINE PREPARATORY COURSE BY NASA KHAN …

© Notes by Dr. Sakib (MRCS Online Preparatory Course By Nasa Khan)

© Notes by Dr. Sakib (MRCS Online Preparatory Course By Nasa Khan) Page | 19

A) 20 mls

B) 50 mls

C) 150 mls

D) 500 mls

E) 1000 mls

77. You are an orthopaedic SHO in clinic. Your consultant, realising that you are about to sit your

examinations, calls you in to see a patient he has just seen. The patient has sustained an injury to a nerve in

his lower limb some time ago. When you examine the patient in front of your consultant. You note loss of

muscle bulk in the anterior and lateral compartments of the affected leg. There is loss of ankle dorsiflexion

and toe extension. The foot itself looks normal. Which is the affected nerve?

A) Femoral nerve

B) Peroneal nerve

C) Popliteal nerve

D) Sciatic nerve

E) Tibial nerve

78. An otherwise healthy 32-year-old man was the driver of a car involved in a high speed RTA 3 days ago.

He has sustained a closed fracture of his femur which has been treated surgically with an intramedullary nail,

as well as fractures of his right clavicle and left radius. He was managed according to ATLS protocol when

he attended the emergency department. On examination, he is acutely short of breath and has a

temperature of 37.5°C. The patient seems confused when you speak to him, and as you examine him, you

note petechial haemorrhages. What do you think is the most likely diagnosis?

A) Asthma attack

B) Chest infection

C) Fat embolism

D) Pulmonary embolism

E) Tension pneumothorax

79. A 27-year-old female with Adult Respiratory Distress Syndrome (ARDS) is ventilated on Intensive Care.

Her inspired oxygen is 100%, Positive End Expiratory Pressure is 15cmH2O and Peak Airway Pressure is

40cmH2O. Her arterial blood gas shows:

PaO2 :6 kPa (11.3-12.6)

PaCO2 :6.9 kPa (4.7-6.0)

SpO2 :88% (>92%)

Which further ventilatory strategy is appropriate in this case?

A) Intravenous Oxygenation (IVOX)

B) Inhaled nitric oxide therapy

C) High Frequency Oscillatory Ventilation (HFOV)

D) Prone position

E) Increasing tidal volume and respiratory rate on the ventilator

80. A 69-year-old man with hypertension is admitted with severe abdominal pain and a pulsatile mass. His

blood pressure is 80/50 mmHg, a pulse of 110 bpm and capillary refill time is 3s, His urea and electrolytes

show:

Page 20: MRCS PART-A ONLINE PREPARATORY COURSE BY NASA KHAN …

© Notes by Dr. Sakib (MRCS Online Preparatory Course By Nasa Khan)

© Notes by Dr. Sakib (MRCS Online Preparatory Course By Nasa Khan) Page | 20

Sodium :135 mmol/L (137-144)

Potassium : 4.9 mmol/L (3.5-4.9)

Urea :18.2 mmol/L (2.5-7.5)

Creatinine :300 µmol/L (60-110)

Aims of resuscitation should be:

A) Immediate intubation and ventilation

B) Restoration of circulation to a normal systolic blood pressure to preserve kidney function

C) Restoration of normal Mean Arterial Pressure

D) Resuscitation before theatre

E) Transfer and stabilisation on HDU

81. You are asked to see a 32-year-old man diagnosed with appendicitis who is awaiting theatre, He has a

pulse of 130 bpm, a blood pressure 105/60 mmHg, respiratory rate 32/min, and his SpO2 is 96%. He is in a

lot of pain. Which of the following is the best option for managing this patient?

A) Catheterisation

B) Blood 1 unit stat

C) Expedite appendicectomy

D) Morphine 10mg intramuscularly

E) Oxygen 100% by face mask

82. 67-year-old man who is 3 days post-operation for a sigmoid colectomy. He has insulin dependent

diabetes mellitus. He complains of dizziness and faintness. His blood pressure is 80/50 mmHg, his pulse is

110 bpm, he has a respiratory rate 24/min, and he has SpO2 99% on air. His blood glucose is 18 mmol/L

(3.0-6.0 Fasting). His electrocardiogram shows ST depression of 2mm in leads II, III and AVF. Which of the

following is the initial drug therapy for this patient?:

A) Aspirin 300mg

B) Clexane lmg/kg subcutaneously

C) Clopidogrel 75mg

D) Diamorphine 2.5mg

E) Glycerol Tri-Nitrate 800mcg sublingually

83. A 67-year-old man complains of dizziness and faintness. He has insulin dependent diabetes mellitus and

he had a sigmoid colectomy 3 days previously. His blood pressure is 80/50 mmHg, his pulse 110 beats per

min, his respiratory rate 24/min, and he has SpO2 99% on air. His plasma glucose concentration is 18

mmol/l, (3.0-6.0 Fasting). Which of the following is the most appropriate investigation for this patient?

A) Arterial blood gas

B) Chest X-ray

C) Electrocardiogram

D) Serum Lactate

E) Urine ket ones

84. A 79-year-old male with critical ischaemia of his foot is awaiting below knee amputation and has lower

limb pain. He is awake and lucid with normal observations. His Full Blood Count shows:

Haemoglobin :12.0 g/dL (13.0-18.0)

Page 21: MRCS PART-A ONLINE PREPARATORY COURSE BY NASA KHAN …

© Notes by Dr. Sakib (MRCS Online Preparatory Course By Nasa Khan)

© Notes by Dr. Sakib (MRCS Online Preparatory Course By Nasa Khan) Page | 21

White Cell Count :14. 0 x 109/L (4-11 x 10

9)

Platelets :67 x 109/L (150-400 x 10

9)

Which of the following is the best option for pain relief?

A) Diclofenac 50mg per oral

B) Epidural analgesia

C) Femoral nerve block

D) Morphine 10mg intravenously

E) Tramadol 50mg per oral

85. A 74-year-old man with ischaemic heart disease is in the surgical High Dependency Unit following a

Hartmanns procedure. He is in pain. He has drained 100/200/300 mis of blood into his drains in the last 3

hours. His blood pressure is 110/80 mmHg, his pulse105 beats/min, he has a respiratory rate 32/min, SpO2

100% on oxygen by face mask.

Haemoglobin :8.1 g/dL (13.0-18.0)

White Cell Count :4.5 x 109/L (4-11 x 10

9)

Platelets :132 x 109/L (150-400 x 10

9)

Which of the following is the next most appropriate therapeutic intervention?

A) 2 units of packed red blood cells

B) 1000mls of crystalloid stat

C) Morphine 5mg intravenously

D) Reassurance

E) Return to theatre

86. A 64-year-old man is admitted with central epigastric pain. Abdominal X-ray shows a central dilated

bowel loop. Temperature 38.4°C; pulse 130 beats per min; blood pressure 80/50 mmHg; respiratory rate

29/min; SpO2 90 on air. His Full blood count:

Haemoglobin :13.0 g/dL (13.0-18.0)

White cell count :3.2 x 109/L (4-11 x 10

9)

Platelets :108 x 109/L (150-400 x 10

9)

MCV :105 fL (80-96)

Which of the following is the most likely diagnosis?

A) Leaking aortic aneurysm

B) MODS

C) Pancreatitis

D) Septic shock

E) SIRS

87. A 70 year-old male is admitted with haematemesis. He Is currently being treated With warferin for atrial

fibrillation and his INR returns as 10 (<1.4). Which of the following is the most appropriate immediate

treatment of his INR?

A) Cryoprecipitate

B) Fresh frozen plasma

C) Intravenous vitamin K

D) Oral vitamin K

Page 22: MRCS PART-A ONLINE PREPARATORY COURSE BY NASA KHAN …

© Notes by Dr. Sakib (MRCS Online Preparatory Course By Nasa Khan)

© Notes by Dr. Sakib (MRCS Online Preparatory Course By Nasa Khan) Page | 22

E) Recombinant Factor VIII concentrate

88. A 7-year-old female presents with an angulated left elbow following a fall from a swing. Examination

reveals a cold left hand with no brachial, radial or u1nar pulses palpable. A plain radiograph of the elbow

reveals a displaced left humeral supracondylar fracture. Which of the following is the most appropriate initial

management?

A) Backslab plaster and review in fracture clinic

B) Heparinisation

C) Manipulation of the fracture under anaesthetic

D) Open reduction and internal fixation of the fracture

E) Surgical exploration of the left brachial artery

89. A 28-year-old female presents eight weeks post-partum with a painful, swollen, erythematous right

breast. On examination there is fluctuance in the right upper outer quadrant adjacent to nipple areolar

complex. The overlying skin is red and tender. What is the most appropriate management for this patient?

A) Excision of abscess cavity

B) Incision and drainage

C) Intravenous antibiotics

D) Mammograghic guided needle aspiration

E) Ultrasound guided needle aspiration

90. A 51 -year-old female is 4 hours post wide local excision of the left breast and axillary node clearance for

breast cancer. She complains of increasing pain from the left breast. On examination the breast is swollen

and tender. Only one drain has been placed in the axilla. What is the most appropriate management in this

case?

A) Analgesia and Observe

B) Stop low molecular weight heparin

C) Surgical wash out and control and Haemostasis

D) Ultrasound guided needle Aspiration

E) Unguided needle Aspiration

91. A 58-year-old female 4 hours post-total thyroidectomy has developed acute shortness of breath. On

examination the patient is visibly distressed and has stridor. Removing the dressings reveals a large

haematoma deep to the wound. Which of the following is the most appropriate immediate management?

A) Cricothyroidotomy

B) Immediate endotracheal intubation

C) Immediate removal of the skin clips and deep sutures at the bed side

D) Immediate shift to theatre

E) Tracheostomy

92.A 60-year-old male is admitted with a two day history of lower abdominal pain and marked vomiting. On

examination he has abdominal swelling, guarding and numerous audible bowel sounds. What is the

diagnosis?

A) Gallst one ileus.

Page 23: MRCS PART-A ONLINE PREPARATORY COURSE BY NASA KHAN …

© Notes by Dr. Sakib (MRCS Online Preparatory Course By Nasa Khan)

© Notes by Dr. Sakib (MRCS Online Preparatory Course By Nasa Khan) Page | 23

B) Ischaemic colitis

C) Large bowel obstruction

D) Sigmoid volvulus.

E) Small bowel obstruction

93. A middle-aged woman presents with recent changes in her bowel habits. She is investigated as a case of

sporadic colonic carcinoma.

A) APC gene mutation

B) β-catenin suppression

C) Down-regulation of p27

D) K-ras suppression

E) p53 upregulation

94. A 53-year-old male is admitted via the accident and emergency department. His main presenting

complaint is severe upper abdominal pain. During the history taking he says he has lost his job within the last

few months and his alcohol intake has been steadily increasing is a result. He is currently drinking over 1

bottle of vodka a day. His admission bloods show a serum amylase of 902 IU/L (50-150). Which of the

following blood tests is NOT part of the Glasgow criteria, for assessing the severity of an attack of

pancreatitis?

A) Albumin

B) Arterial blood gas

C) Calcium

D) CRP

E) LDH

95.A-70-year-old lady has previously had an ileostomy formed three years ago. She presents again because

of upper abdominal pain, vomiting and absence of any output or flatus from the stoma. A CT reveals free

fluid around the small bowel loops. Adhesions are suspected as a cause for the obstruction and a decision is

made to take the patient to theatre. You are asked to consent the patient. What would you NOT include on

die consent form as a potential risk?

A) Anaesthetic risks

B) Anastamotic leak

C) Haemorrhage

D) Splenectomy

E) Stoma formation

96. A patient needs central venous access for total parenteral nutrition (TPN). Which of the following is the

cleanest site for placement?

A) Left femoral

B) Left internal jugular

C) Right femoral

D) Right internal jugular

E) Right subclavian

Page 24: MRCS PART-A ONLINE PREPARATORY COURSE BY NASA KHAN …

© Notes by Dr. Sakib (MRCS Online Preparatory Course By Nasa Khan)

© Notes by Dr. Sakib (MRCS Online Preparatory Course By Nasa Khan) Page | 24

97. A ten-month-old male baby with haemophilia is brought to the Accident and Emergency by his parents

with a 36-hour history of intermittent episodes of inconsolable crying and vomiting. The parents say the

baby’s stools are mixed with blood. On examination, a sausage-shaped mass is palpable over the right side

of abdomen. Per rectal examination reveals an empty rectum but blood is noticed in the glove of the

examining finger.From the options below choose the ONE which you think is the most likely diagnosis in this

patient:

A) Infantile hypertrophic pyloric stenosis

B) Intestinal atresia

C) Intussusception

D) Meckel's diverticulum.

E) Meconium ileus

98. A 44-year-old cyclist is brought to the Accident and Emergency department after he was hit by a car in a

country road. On examination, he appears dyspnoeic and restless. His blood pressure is 106/78 mmHg,

pulse rate is 116/min and respiratory rate is 20/min. On examination of his chest, the trachea is pushed to

the right, the left lung is hyper-resonant to percussion, and there is reduced air-entry in the left lung fields.

From the options below choose the ONE which you think is the most likely diagnosis in this patient:

A) Cardiac tamponade

B) Diaphragmatic rupture

C) Aortic rupture

D) Tension pneumothorax

E) Flial chest

99. A 71-year-old woman presents to the Accident and Emergency department with a three month history of

vague lower abdominal pain, abdominal distension and absolute constipation. On further questioning she

says that she has noticed some blood in her stools, change in bowel habits and a sense of incomplete

evacuation of the bowel. She has lost 2 st ones in weight in the past three months. On examination, the

abdomen is soft but a mass is felt over the left iliac fossa. Plain abdominal X-ray reveals a markedly dilated

colon. From the options below choose the one which you think is the most likely diagnosis in this patient:

A) Crohn's disease

B) Diverticular disease

C) Left-sided colonic carcinoma

D) Sigmoid volvulus

E) Ulcerative colitis

100. A 56-year-old lady who is on long-term non-steroidal drug treatment for arthritis is brought to the

Accident and Emergency department with a 12-hour history of sudden onset epigastric pain, nausea and

vomiting. On examination, she is pale, anxious and lies still. Her blood pressure is 110/80 mmHg and her

pulse rate is 108/min (regular). Abdominal examination reveals severe tenderness over the epigastric region,

generalised guarding and a board like rigidity. An erect chest X-ray reveals gas under the diaphragm. From

the options below choose the ONE which you think is the most likely diagnosis in this patient:

A) Acute cholecystitis

B) Acute pancreatitis

C) Adhesive small bowel obstruction

D) Perforated peptic ulcer

Page 25: MRCS PART-A ONLINE PREPARATORY COURSE BY NASA KHAN …

© Notes by Dr. Sakib (MRCS Online Preparatory Course By Nasa Khan)

© Notes by Dr. Sakib (MRCS Online Preparatory Course By Nasa Khan) Page | 25

E) Ruptured left dome of diaphragm

101. A 34-year-old woman presents to the Accident and Emergency department with a 2-day history of

severe right-sided abdominal pain, increased temperature associated with chills and rigors. She also

complains of burning micturition. She had underg one cystoscopy six days ago. On examination, her

temperature is 39.3º C, has warm and sweaty peripheries, and is tender over her right loin region. Urine

microscopy reveals pus cells and casts.From the options below choose the ONE which you think is the most

likely diagnosis in this patient:

A) Acute glomerul onephritis

B) Goodpasture syndrome

C) Hydroneprosis

D) Pyelonephritis

E) Urinary tract infection

102. A 43-year-old gentleman presents, to the Accident and Emergency unit with severe stabbing type of

pain over the epigastric region. The pain started following episodes of repeated vomiting and it worsens on

swallowing. He is a heavy drinker and has been consuming 10- 15 units of alcohol everyday for the past 20

years. On examination, his blood pressure is 100/78 mmHg, and his pulse is 124/min and thready. Chest x-

ray reveals air in the mediastinum and in the subcutaneous tissues. There is no change in the anatomical

contour of the diaphragm. From the options below choose the most appropriate cause for this patient's signs

and symptoms:

A) Acute cholecystitis

B) Boerhaave's syndrome

C) Diaphragmatic rupture

D) Myocardial contusion

E) Traumatic haemothorax

103. A 43-year-old woman presents to the Accident and Emergency department with an 8-hour history of

severe right upper -quadrant pain and vomiting. She says that the pain exacerbates on breathing and is

radiating to her right scapula. On examination, she is pale and mildly jaundiced. Her pulse rate is 104/min

and temperature is 38.8º C. Abdominal examination reveals a tender mass over the right hypochondrium.

She says that she used to get this type of pain following a fatty meal, although not this severe. Serum

amylase is not elevated. From the options below choose the ONE which you think is the most likely

diagnosis in this patient:

A) Acute appendicitis

B) Acute cholecystitis

C) Acute pancreatitis

D) Cirrhosis of the liver

E) Gall stones

104. A 77-year-old man is brought to the Accident and Emergency department with a 2-hour history of

severe peri-umbilical pain radiating to the back. On examination, he is anxious, pale, sweaty and mildly

cyanosed. His pulse rate is 126/min and blood pressure is 80/60 mmHg. Abdominal examination reveals a

slightly tender mass just to the left of the umbilicus. Bowel sounds are normal. His bilateral femoral pulses

are very feeble. From the options below choose the ONE which you think is the most likely diagnosis in this

patient:

Page 26: MRCS PART-A ONLINE PREPARATORY COURSE BY NASA KHAN …

© Notes by Dr. Sakib (MRCS Online Preparatory Course By Nasa Khan)

© Notes by Dr. Sakib (MRCS Online Preparatory Course By Nasa Khan) Page | 26

A) Acute diverticulitis

B) Acute pancreatitis

C) Adhesive small bowel obstruction

D) Perforated sigmoid colon

E) Ruptured abdominal aortic aneurysm

105. A 53-year-old pedestrian is brought to the Accident and Emergency department after he was hit on a

crossing by a car travelling at about 30 mph. He has numerous bruises and lacerations over his left chest

wall. On examination, he appears pale and anxious. His blood pressure is 90/64 mmHg, heart rate is

116/min, respiratory rate is 20/min and his jugular venous pressure is elevated. Trachea is central and

breath sounds are normal bilaterally but the heart sounds are muffled. A chest radiograph shows fractured

4th, 5th and 6th ribs on the left side and an enlarged cardiac silhouette.From the options below choose the

ONE which you think is the most likely diagnosis in this patient:

A) Traumatic haemothorax

B) Ruptured thoracic aorta

C) Cardiac tamponade

D) Diaphragmatic rupture

E) Tension pneumothorax

106. A-65-year-old man is transferred from the local psychiatric hospital as an emergency and is said to be

suffering with abdominal pain and constipation. He is unable to provide a meaningful history and the only

notes supplied are a drug chart containing a number of psychiatric medications and laxatives. Examination

reveals a distended but minimally tender abdomen with a hyper-resonant percussion note. What is the

diagnosis?

A) Caecal volvulus

B) Colonic carcinoma

C) Diverticular disease

D) Sigmoid volvulus

E) Small bowel obstruction

107. A 45-year-old male presented to the A&E department with a colicky abdominal pain, vomiting and a

painful groin lump. On examination, he had a pulse of 110 bpm, and temperature of 37.8ºC. There was

marked abdominal distension, high-pitched bowel sounds and an erythematous, tender mass in the left groin

above and medial to the pubic tubercle. White cell count 14.7 x109/L (4-11 x10

9) What is the next step in

management?

A) Apply local pressure to the swelling to reduce it

B) Elective surgery

C) Emergency surgery

D) Nil by mouth, naso-gastric tube and intravenous drip

E) Reassurance and OPD appointment

108. A 65-year-old man presents as a emergency with a sudden onset tender mass in the left lower

quadrant. His past medical history includes valve replacement for rheumatic heart disease and he takes

regular warfarin. He has also recently been on a course of erythromycin for a chest infection as he has a

Page 27: MRCS PART-A ONLINE PREPARATORY COURSE BY NASA KHAN …

© Notes by Dr. Sakib (MRCS Online Preparatory Course By Nasa Khan)

© Notes by Dr. Sakib (MRCS Online Preparatory Course By Nasa Khan) Page | 27

penicillin allergy. On examination there is a tender mass to the left of the midline which appears to become

more prominent on lifting the patients head from the bed. Investigations show:

Haemoglobin 8.9 g/dL (12-16)

INR 7.2 (<1.4)

A CT abdomen is performed. Report is awaiting.

What is the likely diagnosis?

A) Carcinoma of Colon

B) Diverticular Abscess

C) Lymphoma

D) Rectus Sheath Haematoma

E) Spigelian Hernia

109. A 24-year-old man is referred to you from the A&E unit. When you attend casualty he is sitting forward

holding his abdomen and complaining of severe epigastric pain. He has vomited on several occasions. On

examination he has marked epigastric tenderness and an amylase requested by the casualty officer returns

as 2100 IU/L and you suspect acute pancreatitis. You ph one the registrar who is in clinic and he asks for the

Imrie/Glasgow score.Which of the following factors is not relevant to the score?

A) Albumin

B) Amylase

C) Lactate dehydrodgenase

D) Urea

E) White cell count

110. A 55-year-old gentleman is admitted with central colicky abdominal pain. The pain has been present for

6 days and is getting worse. He has also noticed the passage of watery diarrhoea for 3 days prior to

presentation. Past medical history includes angina and peripheral vascular disease and he is a current

smoker. Examination reveals generalised tenderness which is most marked over the left colon. An

abdominal x-ray study is performed: What is the likely diagnosis?

A) Colonic carcinoma

B) Crohn's disease

C) Ischaemic colitis

D) Infective colitis

E) Ulcerative colitis

111. A 40-year-old male complains of swelling in the neck. What is the likely diagnosis?

A) Branchial cyst.

B) Carotid artery aneurysm.

C) Carotid body tumour.

D) Lymphocoele

E) Necrotic lymph node.

112. A 5-year-old male has presented to the emergency department with diarrhoea and vomiting of 2 days

duration, which has recently stopped. His normal body weight is 18kg. He has been resuscitated with

Page 28: MRCS PART-A ONLINE PREPARATORY COURSE BY NASA KHAN …

© Notes by Dr. Sakib (MRCS Online Preparatory Course By Nasa Khan)

© Notes by Dr. Sakib (MRCS Online Preparatory Course By Nasa Khan) Page | 28

intravenous fluids but requires intravenous crystalloid maintenance fluids to be prescribed as he refuses oral

intake. Which of the following is the normal maintenance fluid rate per hour in crystalloids for this patient?

A) 36 ml / hour

B) 56 ml / hour

C) 76ml / hour

D) 540 ml / day

E) 1500 ml / day

113. In relation to the oxygen dissociation curve, which of the following factors shifts the curve to the right?

A) Carbon monoxide poisoning

B) Fall in 2,3 DPG

C) Fall in PH

D) Fall in temperature

E) Rise in PH

114. A four-year-old child is admitted to A&E. The child has been rushed to the unit by her grandparents who

are looking after the child, as the parents are at a friend's wedding overseas, and cannot be contacted. The

child is examined and investigations are performed. A provisional diagnosis of a possible ruptured appendix

is made, and the surgical team are called. After further review, the surgical team request to speak to the

child's parents in order to obtain their authority to perform surgery. The child continues to deteriorate. What is

the most appropriate action in this case?

A) The surgeon declines to take the child to theatre, as they are unable to obtain parental

authorisation

B) The surgeon accepts authorisation from the grandparents who state that they do not have

parental responsibility

C) The surgeon performs surgery without authorisation in the child's best interests

D) The surgeon speaks with the charge nurse in A&E who signs the consent form

E) Surgeon, asks that, as the parents are unable to be contacted that a call is made to social

services

115. A baby is admitted on the neonatal unit. When making up an infusion, it is noted that the weight of the

baby has been entered on the drug chart as 13.7kg, but is known to be a premature baby, who is 4 days old.

The baby is reviewed and the observation chart checked. It is confirmed that the weight should be 1.37kg. It

is noted that a number of infusions are already running.What is the most appropriate action in this case?

A) Change the weight on the drug chart, crossing out the incorrect weight

B) Leave the unit immediately as you had no previous involvement in the case

C) Promptly put matters right regarding drugs being administered to the baby then discuss your

concerns with a senior colleague and report the incident

D) Speak with the nurse looking after the baby and change the rate of the infusions running

E) Speak with the parents of the baby and inform them that the baby has had a reaction to the

drugs

Page 29: MRCS PART-A ONLINE PREPARATORY COURSE BY NASA KHAN …

© Notes by Dr. Sakib (MRCS Online Preparatory Course By Nasa Khan)

© Notes by Dr. Sakib (MRCS Online Preparatory Course By Nasa Khan) Page | 29

116. This nerve is a direct continuation of the posterior cord of the brachial plexus. It receives contributions

from C5, C8 and TI. It exits the axilla by leaving the posterior wall, tranversing the triangular space of the

arm.

A) Axillary nerve

B) Median nerve

C) Long thoracic nerve

D) Lower subscapular nerve

E) Radial nerve

117. A 44-year-old man was involved in a head on car crash at 70 mph during which his car burst into

flames. On admission he has a BP 96/65 mmHg; heart rate 130/min regular and has oliguria. After initial

resuscitation his injuries are noted to be: 40% burns; compound fracture of the left tibia; and fracture

dislocation of the right hip. His baseline investigations of note include:

Hb :7.5 g/dL (13.0-18.0)

pH :7.1 (7.36-7.44)

Potassium :6.1 mmol/L (3.5-4.9)

Which of the following is an explanation for his potassium level?

A) Insulin therapy

B) Rhabdomyolysis

C) Steroid therapy

D) SIADH

E) Vomiting

118. A motorcyclist is involved in an RTA and sustains a closed fracture of his left tibia. This is stabilised with

an external fixator. You are called to see him some hours later because of unbearable pain in his leg. You

use a manometer to measure the pressures in his muscle compartments and the reading in the extensor

compartment is 40mmHg. He is taken to theatre for emergency fasciotomy. Which of the following would you

encounter in the extensor compartment?

A) Deep peroneal nerve

B) Peroneus brevis

C) Peroneus longus

D) Sural nerve

E) Tibial nerve

119. A 45-year-old male is found to have a 2cm right adrenal adenoma which was noted coincidentally

following abdominal CT scan performed for investigation of abdominal pain. There are no abnormalities on

examination and the patient is quite well with a blood pressure of 122/84 mmHg. Urine catecholamines are

normal, urine free cortisol normal and plasma renin activity: aldoster one ratio is normal. Which is the most

appropriate management step for this patient?

A) Arrange PET scan

B) Arrange adrenalectomy

C) Characterise further with MRI

D) Repeat imaging in 6 months

E) Reassure and discharge

Page 30: MRCS PART-A ONLINE PREPARATORY COURSE BY NASA KHAN …

© Notes by Dr. Sakib (MRCS Online Preparatory Course By Nasa Khan)

© Notes by Dr. Sakib (MRCS Online Preparatory Course By Nasa Khan) Page | 30

120. A 42-year-old man is seen in the clinic reporting lethargy, nausea and vomiting and weight loss. On

examination he is hypotensive and there is pigmentation of his palmar skin creases and of an old

appendicectomy scar. Routine biochemistry reveals:

Potassium 5.8 mmol/L (3.5-4.9)

Sodium 131 mmol/L (137-144)

What is the likely diagnosis?

A) Addison's disease

B) Conn's syndrome

C) Cushing's syndrome

D) Hypopituitarism.

E) Waterhouse-Friderichsen syndrome

121. Which of the following structures would be seen as a posterior relation of the first part of the duodenum

on a CT scan?

A) Abdominal aorta

B) Main pancreatic duct

C) Portal vein

D) Superior mesenteric vessels

E) Transverse colon

122. During an endoscopy, as the tip of the endoscope descends through the thorax, which of the following

structures are on the left hand side of the oesophagus?

A) Azygous vein

B) Left bronchus

C) Thoracic duct

D) Thoracic vertebra

E) Trachea

123. A 65-year-old man is undergoing surgery for a tumour of his stomach. Before dissection can start, the

surgical anatomy has to be identified. You identify the communicating cavity between the greater and lesser

sacs in the abdomen. Which of the following forms the lower boundary of this area?

A) First part of duodenum

B) Hepatic artery

C) Portal vein

D) Quadrate lobe of the liver

E) Second part of the duodenum

124. An obese 43-year-old lady is undergoing a laparoscopic cholecystectomy. Access is extremely difficult,

and the operation is hampered by bleeding which seems to be a result of arterial damage. The operating

surgeon converts to an open procedure. The bleeding is presumed to be coming from the cystic artery. To

gain temporary haemostasis, the surgeon decides to press on the hepatic artery. Where should he press?

Page 31: MRCS PART-A ONLINE PREPARATORY COURSE BY NASA KHAN …

© Notes by Dr. Sakib (MRCS Online Preparatory Course By Nasa Khan)

© Notes by Dr. Sakib (MRCS Online Preparatory Course By Nasa Khan) Page | 31

A) Anterior wall of the foramen of Winslow

B) Caudate lobe of the liver

C) First part of the duodenum

D) Fundus of the gallbladder

E) Posterior wall of the foramen of Winslow

125. An 18-year-old boy was admitted with RIF pain. On a recent visit to the dentist he had problems with

bleeding after a tooth extraction. A clotting screen was requested which showed a prolonged APTT, normal

PT and bleeding time. A further coagulation screen showed low levels of factor VIII.

A) Haemophilia A

B) DIC

C) Ehlors-Danlos Syndrome

D) Haemophilia B

E) Von Willebrand Disease

126. A 30-year-old gentleman was in an industrial accident leading to 60% burns requiring intensive care

and surgery. A coagulation screen revealed a prolonged APTT, PT and low levels of fibrinogen.

A) Haemophilia A

B) DIC

C) Haemophilia B

D) Von Willebrand Disease

E) Vit K Deficiency

127. A 65-year-old gentleman was admitted with a two month history of jaundice, pale stool, dark urine and

bruising following minimal trauma. A coagulation screen revealed a prolonged APTT and PT.

A) Haemophilia A

B) DIC

C) Ehlors-Danlos Syndrome

D) Von Willebrand Disease

E) Vit K Deficiency

128. A 20-year-old Caucasian man presents with a 6 month history of upper abdominal pain and leg ulcers.

He was noted to be jaundiced by his family. Investigations showed a slight rise in bilirubin and mild anaemia,

An ultrasound scan of the abdomen showed evidence of splenomegaly. What is the likely diagnosis?

A) Sickle cell anaemia

B) Hereditary spherocytosis

C) Iron Deficiency anaemia

D) Sidroblastic anaemia

E) Thalassaemia

129. A 30-year-old gentleman of African origin was admitted with severe back pain. Investigations revealed

a normocytic normochromic anaemia with a high reticulocyte count

Page 32: MRCS PART-A ONLINE PREPARATORY COURSE BY NASA KHAN …

© Notes by Dr. Sakib (MRCS Online Preparatory Course By Nasa Khan)

© Notes by Dr. Sakib (MRCS Online Preparatory Course By Nasa Khan) Page | 32

A) Sickle cell anaemia

B) Iron Deficiency anaemia

C) Sidroblastic anaemia

D) B12 deficiency

E) Pernicious anaemia

130. A 60-year-old gentleman is transfused two units prior to surgery. Within thirty minutes of its

commencement he starts complaining of feeling generally unwell with back pain and shortness of breath.

Observations reveal hypotension, reduced saturations and pyrexia.

A) Air Embolism

B) Thrombophlebitis

C) Haemolytic reaction

D) Hyperkalaemia

E) Iron overload

131. A 75-year-old lady who has had four units of blood prior to surgery is found to have peaked T waves

and a slightly prolonged QRS complex on an ECG.

A) Thrombophlebitis

B) Anaphylaxis

C) Haemolytic reaction

D) Hyperkalaemia

E) Iron overload

132. An 80-year-old lady with a history of ischaemic heart disease is transfused three units, Following this

she starts complaining of increasing shortness of breath. It is noted that her saturations have dropped and

on examination she had bilateral basal crackles.

A) Air Embolism

B) Thrombophlebitis

C) Anaphylaxis

D) Hyperkalaemia

E) Circulatory overload

133. A 70-year-old gentleman was admitted with difficulty swallowing, abdominal pain and vomiting. Past

medical history included a gastrectomy for carcinoma. Initial investigations showed a macrocytic anaemia

and thrombocytopenia.

A) Sickle cell anaemia

B) Iron Deficiency anaemia

C) Sidroblastic anaemia

D) B12 deficiency

E) Pernicious anaemia

134. A 84-year-old man is admitted on the surgical intake complaining of jaundice, weight loss, dark urine

and urticaria. He drinks 30 units of alcohol per week. On examination he is found to have a palpable

gallbladder. Admission bloods include the following LFTs:

Page 33: MRCS PART-A ONLINE PREPARATORY COURSE BY NASA KHAN …

© Notes by Dr. Sakib (MRCS Online Preparatory Course By Nasa Khan)

© Notes by Dr. Sakib (MRCS Online Preparatory Course By Nasa Khan) Page | 33

Bilirubin :156 µmol/L (1-22)

ALT :68 U/L (5-35)

Alkaline Phosphotase :1896 U/L (45-105)

What is the most likely diagnosis from the list below?

A) Cholangiocarcinoma

B) Cholecystitis

C) Cholelithiasis

D) Liver cirrhosis

E) Pancreatic carcinoma

135. A 75-year-old lady presents on the acute general surgical take with severe left iliac fossa pain.

Examination reveals a pyrexia and localised tenderness in the left iliac fossa. She is treated conservatively

with broad spectrum antibiotics and settles over the next 1 week. On review in the out-patient department 3

months later she reports that she has had recurrent urinary tract infections and has pneumaturia.What is the

most likely underlying diagnosis?

A) Colorectal cancer

B) Diverticular disease

C) Ulcerative colitis

D) Crohn's disease

E) Fistula in ano

136. Which among the following statements concerning the lymphatic drainage of the colon is incorrect?

A. Lymph from the intermediate mesocolic lymph nodes drain to the principal nodes

B. Lymph from the caecum drains into the principal nodes at the origin of the inferior mesenteric artery

C. Lymph from the ascending colon passes to the superior mesenteric lymph nodes

D. Lymph from the descending colon passes to the intermediate colic lymph nodes along the left colic artery

E. Lymph from the transverse colon passes to the lymph nodes that lie along the middle

colic artery

137. Which of the following statements regarding renal function is correct?

A. The optimum urine output in an adult in the postoperative period should be 0.1 mL/kg/hour

B. Oliguria in the postoperative period is defi ned as a urine output of less than 25 mL/hour for 2 consecutive

hours

C. Intravenous furosemide should be promptly instituted in a postoperative surgical patient with low urine

output for more than 3 hours

D. A rise in serum creatinine is one of the earliest signs of impending renal failure

E. Pre-existing renal disease is an important cause for postoperative renal dysfunction

138. Which of the following is not a histological feature of malignant tumours?

A. Increased mitosis

B. Pleomorphism

C. Hyperchromatism

D. A decrease in the nuclear:cytoplasmic ratio

E. Focal areas of haemorrhage and necrosis

Page 34: MRCS PART-A ONLINE PREPARATORY COURSE BY NASA KHAN …

© Notes by Dr. Sakib (MRCS Online Preparatory Course By Nasa Khan)

© Notes by Dr. Sakib (MRCS Online Preparatory Course By Nasa Khan) Page | 34

139. A 5-year-old girl is thrown off a swing and lands on her right leg,which becomes tender and swollen.

She is brought to the Emergency Department where radiography demonstrates a fracture of the distal tibia,

which seems to be through a well-defined radiolucent area with sclerotic edges. What is the most likely

diagnosis?

A. Bone cysts

B. Bone metastasis

C. Chondrosarcoma

D. Ewing’s tumour

E. Osteosarcoma

140. A 70-year-old lady presents to the Emergency Department with sudden-onset epigastric pain. Her blood

pressure is 110/80 mmHg and her pulse rate is 110/min (regular). The only ailment in her medical history is

osteoarthritis. Plain abdominal radiography is normal. An erect chest radiograph shows gas under the

diaphragm. A rectal examination reveals melaena. What is the most likely diagnosis?

A. Ureteric colic

B. Crohn’s disease

C. Adhesive small bowel obstruction

D. Perforated peptic ulcer

E. Acute pancreatitis

141. Which of the following statements is incorrect regarding the physiological effects following

trauma/surgery?

A. A raised basal metabolic rate results in proteolysis in the absence of calorifi c intake

B. In sepsis or shock, hypoglycaemia may ensue in the diabetogenic state

C. Serum ketone levels are usually high

D. Proteolytic states result in a negative nitrogen balance

E. There is a tendency towards sodium retention

142. A 38-year-old man sustains major trauma to multiple organs in a road traffic accident. He is admitted to

intensive care, where he is fully ventilated. His presentation is soon complicated by septic shock, for which

he is commenced on noradrenaline, continuous veno-venous haemofiltration and intravenous antibiotics. His

mean arterial pressure remains at 60 mmHg and does not improve after his noradrenaline is changed to

adrenaline. There is no evidence of myocardial dysfunction. What would be the most appropriate next step in

managing this patient?

A. Activated protein C

B. Change of inotropes

C. Dexamethasone

D. Hydrocortisone

E. Nitric oxide

143. A 34-year-old secretary presents to the Emergency Department after sustaining multiple penetrating

injuries to her right upper limb. Careful examination reveals no vascular injury to the limb, and sensory

function remains intact. On examination of motor function, however, finger extension is severely limited.

Which nerve is most likely to have been damaged?

A. Anterior interosseous nerve

B. Medial antebrachial cutaneous nerve

C. Musculocutaneous nerve

Page 35: MRCS PART-A ONLINE PREPARATORY COURSE BY NASA KHAN …

© Notes by Dr. Sakib (MRCS Online Preparatory Course By Nasa Khan)

© Notes by Dr. Sakib (MRCS Online Preparatory Course By Nasa Khan) Page | 35

D. Posterior interosseous nerve

E. Radial nerve

144. Which of the following is not true about insulin?

A. It increases glucose uptake in tissues

B. It increases the uptake of amino acids and lipids in tissues

C. It stimulates glycogenesis

D. It increases protein catabolism

E. It stimulates lipid oxidation

145. A 60-year-old smoker with apical lung cancer has signs and symptoms of Horner’s syndrome. Which of

the following is not a feature of Horner’s syndrome?

A. Ptosis

B. Miosis

C. Enophthalmos

D. Loss of ciliospinal refl ex

E. Loss of lacrimation

146. A 5-year-old boy presents to his GP with a 1-week history of progressively worsening left hip pain and

limping. The GP suspects Perthes’ disease and reassures the boy and his parents. Which imaging modality

is most likely to detect the earliest changes of Perthes’ disease?

A. Plain radiography

B. Computed tomography

C. Magnetic resonance imaging

D. Ultrasonography

E. Radioisotope bone scanning

147. A 45-year-old diabetic man is referred to the Emergency Department by his GP for recurrence of a

previously drained perianal abscess. He is admitted for examination under anaesthesia, which reveals the

presence of a fistula draining the abscess cavity. The abscess is clearly visible at 7 o’clock in the lithotomy

position. What is the most likely location of the internal opening of the fistula

tract?

A. At 12 o’clock (i.e. anterior to the anus)

B. At 3 o’clock (i.e. left of the anus)

C. At 6 o’clock (i.e. posterior to the anus)

D. At 7 o’clock (i.e. the fi stula tract is in a straight line from the abscess cavity)

E. At 9 o’clock (i.e. right of the anus)

148. A 30-year-old female undergoes a total pancreatectomy for severe necrotizing pancreatitis that does

not respond to conservative therapy. Which of the following statements is incorrect regarding the

physiological effects of total pancreatectomy on this patient?

A. Protein malnutrition and negative nitrogen balance ensues from a loss of proteolytic proenzymes

B. The patient is likely to develop diabetes mellitus

C. Loss of fat emulsifi cation will result in the malabsorption of vitamins A, C, D, and K

D. The patient is at a higher risk of developing iron defi cient anaemia

E. The patient has a higher risk of developing osteoporosis

Page 36: MRCS PART-A ONLINE PREPARATORY COURSE BY NASA KHAN …

© Notes by Dr. Sakib (MRCS Online Preparatory Course By Nasa Khan)

© Notes by Dr. Sakib (MRCS Online Preparatory Course By Nasa Khan) Page | 36

149. Which of the following is not a necessary topic when obtaining informed consent from a patient before

an invasive investigation or intervention?

A. Explanation of the likely benefi ts and the probabilities of success

B. The purpose of a proposed investigation or treatment

C. Advice about whether the proposed treatment is experimental

D. Comparative figures of the surgeon’s rate of complications against the national rate of complications

E. Other options for investigation or management of the condition

150.. A urologist performs a radical cystectomy for locally advanced bladder malignancy in an elderly

gentleman. Which of the following statements about the anatomy of the urinary bladder is correct?

A. A peritoneal fold separates the bladder from the pubic symphysis

B. The bladder is related superomedially to the levator ani muscle

C. The external iliac veins drain the venous plexus of the bladder

D. The transitional cell epithelium of the bladder is derived from ectoderm

E. The bladder is located in the abdomen in young children

151. Which of the following statements is not true regarding the secretion of antidiuretic hormone (ADH)?

A. Osmoreceptors situated in the hypothalamus are sensitive to increasing plasma osmolarity

B. Baroreceptors situated in the atria of the heart are sensitive to circulating blood volume

C. Stretch receptors situated in the carotid arteries are stimulated when the blood pressure rises

D. Head injury stimulates ADH secretion

E. Prolonged hypoxia can lead to oliguria and hyponatraemia

152. A 70-year-old male presents with lower urinary tract symptoms. A diagnosis of benign prostatic

hyperplasia is made. Which of the following could be a likely presenting feature in the patient?

A. Phimosis

B. Enlarged posterior lobes of prostate

C. Retrograde ejaculation

D. Vaginal hydrocoele

E. Detrusor atony

153. A 79-year-old gentleman with multiple medical comorbidities is undergoing excision of a basal cell

carcinoma from his periorbital region under local anaesthesia. Which of the following medications should

preferably be stopped prior to surgery?

A. Prednisolone

B. Aspirin

C. Propranolol

D. Gliclazide

E. Bendrofl uazide

154. An 83-year-old gentleman with a history of hypertension, hypercholesterolaemia, insulin-dependent

diabetes mellitus and peripheral vascular disease, suffers a transient ischaemic attack

involving temporary weakness of his left arm and left leg. Initial investigations including ECG and

transthoracic echocardiographyare normal. After complete recovery of his motor function, a carotid

duplex ultrasound scan reveals complete stenosis of his left internal carotid artery and a 71% stenosis of his

right internal carotid artery. Which of the following treatment options would be most appropriate for this

patient?

Page 37: MRCS PART-A ONLINE PREPARATORY COURSE BY NASA KHAN …

© Notes by Dr. Sakib (MRCS Online Preparatory Course By Nasa Khan)

© Notes by Dr. Sakib (MRCS Online Preparatory Course By Nasa Khan) Page | 37

A. Right carotid bypass

B. Right carotid endarterectomy

C. Bilateral carotid endarterectomy

D. Left carotid angioplasty

E. Left carotid endarterectomy

155. A 30-year-old male undergoes a right superficial parotidectomy. Eight months later he presents to the

outpatient department complaining of flushing and sweating of the right side of his face on eating. He is

diagnosed as having Frey’s syndrome and is listed for botulinum toxin injections. Misdirected re-innervation

of which nerve is responsible for this syndrome?

A. Greater auricular nerve

B. Facial nerve

C. Trigeminal nerve

D. Auriculotemporal nerve

E. Greater petrosal nerve

156. Which of the following statements is incorrect regarding the action of nitric oxide (NO)?

A. NO causes vasoconstriction by causing contraction of smooth muscle cells

B. NO regulates vascular remodelling

C. NO is responsible for penile erection by causing relaxation of the corpus cavernosum

D. NO prevents platelet aggregation and adhesion

E. NO plays an important function in the relaxation of the anal sphincter

157. A 30-year-old man presents with severe right loin pain radiating to his groin. He undergoes a CT scan

of his kidneys, ureters and bladder, and is diagnosed with a ureteric stone. At which anatomical site may the

stone have been impacted?

A. Pelviureteric junction

B. Point at which the iliac vessels cross the ureter

C. Vesicoureteric junction

D. All of the above

E. None of the above

158. An 85-year-old retired gentleman is referred to the Emergency Department by his GP for a 24-hour

history of melaena. The patient was commenced on warfarin 4 weeks previously, after a new

diagnosis of atrial fibrillation. The gentleman’s blood pressure is 84/59 mmHg, his heart rate is 110/min, and

his respiratory rate is 20/min. Cardiorespiratory and abdominal examinations are otherwise normal; a rectal

examination confirms melaena. Initial blood tests reveal a Hb of 8.5 g/L, MCV of 80 fL, and INR of 6.3. Which

of the following options is most appropriate for managing this patient’s coagulopathy?

A. Fresh frozen plasma only

B. Intravenous vitamin K only

C. Stop warfarin

D. Stop warfarin and administer intravenous vitamin K

E. Stop warfarin and administer intravenous vitamin K with prothrombin complexconcentrate

159. A 42-year-old asymptomatic woman undergoes an ultrasound scan of her abdomen as part of a routine

private health check-up. The ultrasound scan shows gallstones, and she is subsequently referred to the

surgical outpatient clinic. What is the treatment of choice?

A. Laparoscopic cholecystectomy

Page 38: MRCS PART-A ONLINE PREPARATORY COURSE BY NASA KHAN …

© Notes by Dr. Sakib (MRCS Online Preparatory Course By Nasa Khan)

© Notes by Dr. Sakib (MRCS Online Preparatory Course By Nasa Khan) Page | 38

B. Repeat ultrasound

C. No treatment—reassurance and discharge

D. Magnetic resonance cholangiopancreatography (MRCP)

E. Endoscopic retrograde cholangiopancreatography (ERCP)

160. A 30-year-old man with a history of alcoholic liver cirrhosis and portal hypertension presents with

sudden-onset massive haematemesis. Gastroscopy reveals bleeding oesophageal varices, which are then

banded. Which one of the following is true about the anatomy of the portal venous system?

A. The portal vein arises from the confl uence of the splenic and the inferior mesenteric veins

B. The portal vein arises behind the neck of the pancreas.

C. Porto-systemic anastomoses are found only in the lower oesophagus and around the umbilicus.

D. The portal vein drains the GI tract from the upper oesophagus to the anorectal junction

E. The portal vein lies anterior to the common bile duct and hepatic artery in the lesser omentum

161. Which of the following is not true about the cardiac cycle?

A. The cardiac cycle lasts about 0.8 seconds

B. Atrial contraction contributes to about 30% of atrial fi lling

C. During exercise there is disproportionate shortening of ventricular relaxation

D. Isovolumetric contraction is initiated by the QRS complex

E. The T wave corresponds to reduced ventricular ejection

162. A 19-year-old man is brought to the Emergency Department after being found unconscious outside a

pub. On examination he has a GCS of 3 with a blood pressure of 116/84 mmHg and pulse rate of 94/min. He

smells heavily of alcohol. He is noted to have bilateral periorbital haematomas and bruising over the right

mastoid process. There appears to be serosanguinous discharge from his right nostril and right ear.

Otoscopic examination of the right ear reveals blood behind the tympanic membrane. What is the most likely

diagnosis in this patient?

A. Le Fort I fracture

B. Le Fort II fracture

C. Le Fort III fracture

D. Basal skull fracture

E. Extradural haemorrhage

163. A 45-year-old male presents to his GP with groin lumps on both sides. Examination reveals firm bilateral

inguinal lymphadenopathy. The GP attempts to recall the lymphatic drainage of the region in order to

suggest potential causes for this. Which one of the following anatomical structures does not drain to the

superficial inguinal lymph nodes?

A. Perineum

B. Feet

C. Scrotum

D. Testicle

E. Lower anal canal

164. A 55-year-old patient presents to her GP with a longstanding history of nausea. Which of the following

statements is incorrect regarding the mechanisms associated with mediating nausea?

A. Stimulation of chemoreceptors and stretch receptors of the gut are transmitted by the afferent vagal

nerves to produce nausea

B. The chemoreceptor trigger zone senses potentially toxic chemicals in the gut

C. Motion sickness is mediated through vestibular apparatus

Page 39: MRCS PART-A ONLINE PREPARATORY COURSE BY NASA KHAN …

© Notes by Dr. Sakib (MRCS Online Preparatory Course By Nasa Khan)

© Notes by Dr. Sakib (MRCS Online Preparatory Course By Nasa Khan) Page | 39

D. Disequilibrium caused by alcohol is mediated via the vestibular apparatus

E. Chemoreceptor trigger zone modulates complex experiences such as taste, sight, smell, memory, and

emotion involved in anticipatory nausea

165. A 10-year-old boy presents to the Emergency Department with a 24-hour history of progressive right

testicular pain. Examination reveals normal lying testes and some scrotal erythema and swelling. The right

testes, epididymis, and spermatic cord are tender, and Prehn’s manoeuvre leads to some relief of the pain.

What is the most likely diagnosis in this patient?

A. Testicular torsion

B. Epididymitis

C. Epididymo-orchitis

D. Orchitis

E. Idiopathic scrotal oedema

166. A 35-year-old-man is brought to the Emergency Department following a road traffic accident. He

demonstrates signs and symptoms of a simple pneumothorax, which is confirmed on chest radiography.

Which of the following anatomical landmarks is not useful for delineating the 'safe triangle' for chest drain

insertion in this patient?

A. Apex of axilla

B. Inferolateral border of pectoralis major

C. Anterior border of the latissimus dorsi

D. Nipple

E. Mid-clavicular line

167.Which among the following statements concerning atropine is incorrect?

A. It is a muscarinic receptor agonist

B. it leads to an increased heart rate

C. It is mostly excreted in the urine

D. It has a half-life of about 2–3 hours

E. It decreases bronchial and salivary secretions

168. Before dissection of a gastric tumour, a surgeon identifies the communicating cavity between the

greater and lesser sacs in the abdomen. Which of the following forms the inferior boundary of

this area?

A. Caudate lobe of the liver

B. First part of duodenum

C. Gastroduodenal artery

D. Hepatic portal vein

E. Third part of the duodenum

169. A 47-year-old male presents with pruritis and lethargy over the past 6 months. On examination he is

mildly jaundiced and has scratch marks over his lower limbs. Examination reveals a palpable liver edge 2 cm

below the costal margin and mild tenderness in the right upper quadrant. Blood tests reveal a normal FBC,

bilirubin 30μmol/L, ALT 115 U/L, AST 150 U/L, GGT 110 U/L, cholesterol 7.2 mmol/L, negative hepatitis

serology, a negative antinuclear antibody level, and a positive antimitochondrial antibody level. He

undergoes an ultrasound scan which shows diffuse parenchymal changes with no duct dilatation or

gallstones. What is the most likely diagnosis?

A. Cholangiocarcinoma

B. Haemochromatosis

Page 40: MRCS PART-A ONLINE PREPARATORY COURSE BY NASA KHAN …

© Notes by Dr. Sakib (MRCS Online Preparatory Course By Nasa Khan)

© Notes by Dr. Sakib (MRCS Online Preparatory Course By Nasa Khan) Page | 40

C. Primary biliary cirrhosis

D. Primary sclerosing cholangitis

E. Wilson’s disease

170. During open hepatobiliary surgery, the surgeon accidentally injures the structure directly posterior to the

second part of the duodenum whilst attempting to mobilize this segment of bowel. Which one of the following

structures is most likely to have been injured?

A. Fundus of the gallbladder

B. Hepatic portal vein

C. Hilum of the right kidney

D. Superior mesenteric artery

E. Gastric pylorus

171. A 10-month-old baby boy is brought to the Paediatric Surgical Emergency unit by his parents with a 24-

hour history of intermittent episodes of crying, vomiting, and refusal to feed. The parents say that they have

noticed the baby’s stools to be mixed with blood. On examination, a ‘sausage-shaped’ mass is palpable over

the right side of abdomen. Per rectal examination reveals an empty rectum but blood is noticed in the glove

of the examining finger. What is the most likely diagnosis in this patient?

A. Hirschsprung’s disease

B. Intussusception

C. Duodenal atresia

D. Meconium ileus

E. Infantile hypertrophic pyloric stenosis

172. Which of the following nerves does not arise from the posterior cord of brachial plexus?

A. Upper subscapular nerve

B. Long thoracic nerve

C. Thoracodorsal nerve

D. Axillary nerve

E. Radial nerve

173. A 65-year-old man is referred to the colorectal clinic with a 2-month history of faecal urgency, tenesmus,

and diarrhoea with occasionally fresh rectal bleeding. A screening colonoscopy performed 6 months

previously was reported as normal. He has had no recent weight loss or loss of appetite. His past medical

history includes hypertension and prostate cancer, for which he received external beam radiotherapy one

year previously. Abdominal examination is unremarkable and rectal

examination does not reveal fissures, haemorrhoids, or rectal polyps. What is the most likely diagnosis in this

patient?

A. Anal fistula

B. Clostridium difficile proctitis

C. Diverticular disease

D. Radiation proctitis

E. Ulcerative colitis

174. A 45-year-old male presented to the Emergency Department complaining of a fish bone stuck in his

throat. Despite soft tissue radiographs of the neck confirming the presence of the bone, it could not be

detected during examination of the larynx with a laryngeal mirror. Eventually, endoscopic examination of the

laryngopharynx allowed for the bone to be visualized and retrieved. Which structure may have been injured if

the fish bone had pierced the mucous membrane?

A. Aryepiglottic fold

Page 41: MRCS PART-A ONLINE PREPARATORY COURSE BY NASA KHAN …

© Notes by Dr. Sakib (MRCS Online Preparatory Course By Nasa Khan)

© Notes by Dr. Sakib (MRCS Online Preparatory Course By Nasa Khan) Page | 41

B. Internal laryngeal nerve

C. Larynx

D. Recurrent laryngeal nerve

E. Vocal cords

175. A 55-year-old lady with polymyalgia rheumatica is brought to the Emergency Department by ambulance

for sudden onset chest pain, which started while she was driving. The pain was described as being 'ripping'

in nature and radiated to her back and jaw. Her pulse rate is 110/min, and blood pressure from her right and

left arms are 142/70 mmHg and 167/82 mmHg respectively. A portable CXR reveals a widened mediastinum

and a small pleural effusion. What is the most likely diagnosis?

A. Aortic dissection

B. Acute myocardial infarction

C. Boerhaave’s syndrome

D. Cardiac tamponade

E. Leaking abdominal aortic aneurysm

176. Which of the following statements concerning universal precautions in surgical practice is not true?

A. Water-repellent gowns reduce the risk of transmission of blood-borne diseases

B. Wearing rubber gloves does not protect against sharps injury

C. Needles should be re-sheathed before handing back to the scrub nurse

D. Scalpels should be passed in a kidney bowl

E. Fixed retraction devices reduce the risk of injury due to sharp objects

177. A 35-year-old male undergoes a terminal ileal resection for Crohn’s fistulation. Which of the following

conditions is least likely to occur as a result of his operation?

A. Vitamin C defi ciency

B. Gallstones

C. Subacute combined degeneration of the spinal cord.

D. Macrocytic anaemia

E. Diarrhoea

178. An 80-year-old osteoporotic female presents to the Emergency Department after having slipped and

fallen on her outstretched right arm. After she is diagnosed with a Colles’ fracture, she undergoes

manipulation under anaesthesia and is eventually discharged home in a cast. At her two-week follow-up in

the fracture clinic, she is found to be unable to move her thumb away from the rest of her hand (i.e. in the

same plane as her hand), whilst her palm is resting flat on the table. Which of the following structures is she

most likely to have damaged?

A. Abductor pollicis longus

B. Extensor pollicis longus

C. Abductor pollicis brevis

D. Radial nerve

E. Median nerve

179. Which of the following options represents the commonest clinical manifestation of primary

hyperparathyroidism?

A. Bone disease

B. Constipation

C. Pancreatitis

D. Polyuria

Page 42: MRCS PART-A ONLINE PREPARATORY COURSE BY NASA KHAN …

© Notes by Dr. Sakib (MRCS Online Preparatory Course By Nasa Khan)

© Notes by Dr. Sakib (MRCS Online Preparatory Course By Nasa Khan) Page | 42

E. Renal stone disease

180. A 3-year-old boy is brought to the Emergency Department after a television fell onto his right knee. The

child is in a lot of pain and the right knee appears swollen. Radiography of the right knee reveals a fracture of

the distal femur extending through the epiphysis, physis, and metaphysis. Which type of epiphyseal injury

has the child sustained, according to the Salter–Harris classification?

A. Type I

B. Type II

C. Type III

D. Type IV

E. Type V

End of the Paper